Pls answer this!!!!
with simple working out
<333

Pls Answer This!!!!with Simple Working Out&lt;333

Answers

Answer 1
Someone is answering it right now I just really need the points
Answer 2

Answer:

Step-by-step explanation:

A is 40

B 3 on the the top after 9 is 27

term is 3

c is 4 on top after 32 is 128

term is 4

d after 300 is 3,000

term is 10


Related Questions

A.14.5 square inches
B.29 square inches
c.20.5 square inches
d.32 square inches

Answers

Answer:

d. 32 in.²

Step-by-step explanation:

Bottom face: 6 in. × 0.5 inch

Side faces: 2 × 5 in. × 0.5 in.

Front and back faces: 2 x 6 in. × 4 in. / 2

surface area = 3 in.² + 5 in.² + 24 in.²

surface area = 32 in.²

on jerry's text messaging phone plan, he paid $0.60 for every 30 messages he sent. what is the his cost per message?

Answers

Answer:

$0.02

Step-by-step explanation:

0.60/30

Which value will be assigned to z in line 12 under static sexping? (b) Which value will be assigned to 2 in line 12 under dynamic scoping? I might be instructive to draw the runtime stack for different times of the execution. Inut it is not strictly required. Draw the runtime stack after each line executes! Exercise 3. Parameter Passing Consider the following block. Ansune static scaping { int y: int z; - 7 { int (int a) 4 yari: return (yta) 1 int g(int x) { y = f(x+1)+1; 2:- 1( x3): return (z+1) } 2 :- g(y2): : 12 13 14 is) What are the values of y and 2 at the end of the following block under the assumption that both parameters a und x repassed: la) Call-by-Name (h) Calltyy Need It might be instructive to draw the runtime stack for differcut times of the execution, but it is not strictly required Draw the runtime stack after each line executes

Answers

The runtime stack for dynamic scoping at the end of the block would be:

y: f(x+1)+1
z: f(x+1)+1+1

Under static scoping, the value of z in line 12 will be 7. Under dynamic scoping, the value of z in line 12 will be the value of y in line 2, which is equal to f(x+1)+1. The values of y and z in the end of the block will differ depending on the parameter passing method used.

For call-by-name, the value of y at the end of the block will be f(x+1)+1 and the value of z will be f(x+1)+1+1. For call-by-need, the value of y will be f(x+1)+1 and the value of z will be f(x+1)+1+1.

It might be instructive to draw the runtime stack for different times of the execution, but it is not strictly required. The runtime stack for static scoping at the end of the block would be:

y: f(x+1)+1
z: 7


The runtime stack for dynamic scoping at the end of the block would be:

y: f(x+1)+1
z: f(x+1)+1+1Learn more about Dynamicbrainly.in/question/23656646#SPJ11

Don bought the furniture listed below he paid $500 and will make monthly payments of $85 for the remaining amount how long will it take to pay for the furniture

Answers

Answer:

it will take approximately 5.88 months for Don to pay off the remaining amount of $R = $85t = $85(5.88) = $499.80

Step-by-step explanation:

Don paid $500 upfront and will make monthly payments of $85 for the remaining amount. Let's assume the remaining amount he needs to pay is $R. The total cost of the furniture is the sum of the amount paid upfront and the remaining amount:

Total Cost = $500 + $R

Since he will be paying $85 per month, we can set up an equation to determine the time it will take to pay off the remaining amount:

$R = $85t

where t is the number of months it will take to pay off the remaining amount.

Substituting $R = $85t in the total cost equation, we get:

Total Cost = $500 + $85t

Since we want to find the time it will take to pay off the furniture, we need to solve for t. We can equate the total cost to the amount Don will pay at the end of the payment period, which is:

Total Cost = Amount Paid

$500 + $85t = $500 + $85t + $R

$85t = $R

$500 + $85t = $500 + $85t + $85t

$500 + $170t = $500 + $R

$170t = $R

Substituting $R = $85t, we get:

$170t = $85t

t = $500/$85

t = 5.88 (rounded to two decimal places)

Dakota earned ​$6.00 in interest in Account A and ​30.00$ in interest in Account B after months. If the simple interest rate is ​4% for Account A and ​5% for Account​ B, which account has the greater​ principal? Explain.

Answers

the principal in Account B is 4.8 times the principal in Account A.

How to solve?

Let the principal in Account A be P and the principal in Account B be Q. Also, let n be the number of months.

From the given information, we have:

Interest earned in Account A = $6.00

Interest rate in Account A = 4%

Number of months = n

Using the formula for simple interest, we have:

Interest earned in Account A = (P × 4%× n) / 12

Substituting the given values, we get:

6.00 = (P × 4% × n) / 12

P× n = 150

Similarly, we have:

Interest earned in Account B = $30.00

Interest rate in Account B = 5%

Number of months = n

Using the formula for simple interest, we have:

Interest earned in Account B = (Q× 5% × n) / 12

Substituting the given values, we get:

30.00 = (Q× 5% ×n) / 12

Q × n = 720

To compare the principals, we can divide the equation for Account B by the equation for Account A:

(Q × n) / (P× n) = 720 / 150

Simplifying, we get:

Q / P = 4.8

Therefore, the principal in Account B is 4.8 times the principal in Account A.

Since the interest rate in Account B is higher than the interest rate in Account A, we can conclude that the principal in Account B is greater than the principal in Account A.

To know more about Simple Interest visit:

https://brainly.com/question/25845758

#SPJ1

It takes 120 unit cubes to completely fill a rectangular prism. The prism is 10 unit cubes high. What are the other possible dimensions of the rectangular prism?

Answers

the possible dimensions of the rectangular prism are:

1 unit by 12 units by 10 units

2 units by 6 units by 10 units

3 units by 4 units by 10 units

Let the length and width of the rectangular prism be represented by l and w, respectively. Since the height of the prism is 10 unit cubes, we know that the volume of the prism is:

V = l × w × 10

We are also given that the prism requires 120 unit cubes to fill completely, which means:

V = l × w × 10 = 120

Dividing both sides by 10, we get:

l × w = 12

Now we need to find pairs of positive integers l and w that multiply to 12. These are:

1 × 12

2 × 6

3 × 4

Therefore, the possible dimensions of the rectangular prism are:

1 unit by 12 units by 10 units

2 units by 6 units by 10 units

3 units by 4 units by 10 units

Note that these dimensions are not unique, as we can also obtain different dimensions by exchanging the length and width, or by rotating the prism.

To know more about rectangular prism, visit: brainly.com/question/21308574

#SPJ4

If John had 3 apples then Droped 2 then found 4 then gave one to his friend how many apples does he have now​

Answers

Answer:

4 apples

Step-by-step explanation:

We know

John had 3 apples, then Dropped 2, found 4, then gave 1 to his friend.

How many apples does he have now​?

3 - 2 + 4 - 1 = 4 apples

So, he has 4 apples now.

Answer:

4 apples

Step-by-step explanation:

We know that John had 3 apples

but then he had dropped 2 of them

he then found 4

then he gave 1 to his friend

3-2=1

1+4=5

5-1=4

The answer is 4

Hope this helps!

how many yard are in 10 meters

Answers

Answer:

10.936 yards

Step-by-step explanation:

10.936 yards. Is this the answer?

If a first sample has a sample variance of 12 and a second sample has a sample variance of 22 , which of the following could be the value of the pooled sample variance? 1 10 16 25

Answers

The value of the pooled sample variance is 25 when the first sample has a sample variance of 12 and a second sample has a sample variance of 22.

If a first sample has a sample variance of 12 and a second sample has a sample variance of 22, then the possible values of the pooled sample variance are given by the formula below:

Formula:

pooled sample variance = [(n₁ - 1) s₁² + (n₂ - 1) s₂²] / (n₁ + n₂ - 2)

Where s₁ and s₂ are the sample standard deviations of the first and second samples,

n₁ and n₂ are the sample sizes of the first and second samples, respectively.

Thus, substituting the given values into the formula above, we have pooled sample variance:

= [(n₁ - 1) s₁² + (n₂ - 1) s₂²] / (n₁ + n₂ - 2)

= [(n₁ - 1) 12 + (n₂ - 1) 22] / (n₁ + n₂ - 2)

Checking each of the answer options:

If pooled sample variance is 1, then:

(n₁ - 1) 12 + (n₂ - 1) 22

= (n₁ + n₂ - 2)(1)

= 12n₁ + 22n₂ - 34

= (12n₁ - 12) + (22n₂ - 22)

= 12(n₁ - 1) + 22(n₂ - 1)

The expression on the right-hand side of the equation is a sum of multiples of 12 and 22, and therefore, the expression itself will be a multiple of the greatest common divisor of 12 and 22, which is 2.

Since 34 is not a multiple of 2, the equation cannot be true if the pooled sample variance is 1.

Thus, 1 is not a possible value of the pooled sample variance.

If pooled sample variance is 10, then:

(n₁ - 1) 12 + (n₂ - 1) 22

= (n₁ + n₂ - 2)(10)

= 12n₁ + 22n₂ - 34

= (12n₁ - 12) + (22n₂ - 22)

= 12(n₁ - 1) + 22(n₂ - 1)

The expression on the right-hand side of the equation is a sum of multiples of 12 and 22, and therefore, the expression itself will be a multiple of the greatest common divisor of 12 and 22, which is 2.

Since 34 is not a multiple of 2, the equation cannot be true if the pooled sample variance is 10.

Thus, 10 is not a possible value of the pooled sample variance.

If pooled sample variance is 16, then:

(n₁ - 1) 12 + (n₂ - 1) 22

= (n₁ + n₂ - 2)(16)

= 12n₁ + 22n₂ - 34

= (12n₁ - 12) + (22n₂ - 22)

= 12(n₁ - 1) + 22(n₂ - 1)

The expression on the right-hand side of the equation is a sum of multiples of 12 and 22, and therefore, the expression itself will be a multiple of the greatest common divisor of 12 and 22, which is 2.

Since 34 is not a multiple of 2, the equation cannot be true if the pooled sample variance is 16.

Thus, 16 is not a possible value of the pooled sample variance.

If pooled sample variance is 25, then:

(n₁ - 1) 12 + (n₂ - 1) 22

= (n₁ + n₂ - 2)(25)

= 12n₁ + 22n₂ - 34

= (12n₁ - 12) + (22n₂ - 22)

= 12(n₁ - 1) + 22(n₂ - 1)

The expression on the right-hand side of the equation is a sum of multiples of 12 and 22, and therefore, the expression itself will be a multiple of the greatest common divisor of 12 and 22, which is 2.

Since 46 is a multiple of 2, the equation can be true if the pooled sample variance is 25.

Thus, 25 is a possible value of the pooled sample variance.

Learn more about sample variance here:

brainly.com/question/29077187

#SPJ11

Given the functions f(x) = 1/x-1 + 1 and g(x)=1/x+2 +4 describe the transformation of the graph of function f onto the graph of function g

Answers

Transformation of the graph from parent function f(x) to g(x) is:

there is shift of 3 units towards the right to reach from -1 to  2 along x axis.there is a shift of 3 units upward to reach from 1 to 4 on y axis.Explain about the transformation of coordinates?

That whenever a figure is relocated from one place to another without affecting its dimension, shape, or orientation, a transition known as translation takes place. If we are aware of the direction and magnitude of the figure's movement, we may draw the translation in the coordinate plane.

The x - axis and y values are reversed. 180° rotation about the origin Every x and y value reverses from what it was. Each current value becomes the counterpart of what it was after a 270° rotation of the origin. The two x and y values are reversed.

The given function:

f(x) = 1/x-1 + 1 and g(x)=1/x+2 +4

g(x) is obtained function after transformation:

transformation:

there is shift of 3 units towards the right to reach from -1 to  2 along x axis.there is a shift of 3 units upward to reach from 1 to 4 on y axis.

Know more about the transformation of coordinates

https://brainly.com/question/18493427

#SPJ1

10 * x = 425
81729 / y = 898.12

What is X and Y?
I WILL GIVE BRAINLY IF U DONT USE CALCULATOR

Answers

The value of X is 42.5 and Y is 90.98.

How to solve equations?

To solve an equation, you need to perform the same operation on both sides of the equation until you isolate the variable on one side and have a numerical value on the other side. The process of solving an equation generally involves the following steps:

Simplify both sides of the equation by combining like terms and following the order of operations.

Add or subtract the same value from both sides of the equation to isolate the variable term.

Multiply or divide both sides of the equation by the same non-zero value to isolate the variable term.

Check your solution by plugging it back into the original equation and verifying that it satisfies the equation.

Solving the given equations :
To solve for X, we can use inverse operations to isolate the variable. Since 10 is multiplied by X, we can use the inverse operation of division by 10 to isolate X. So, dividing both sides of the equation by 10 gives:

[tex]10x/10 = 425/10[/tex]

Simplifying the left side of the equation gives:

[tex]x = 42.5[/tex]

Therefore, X is 42.5.

To solve for Y, we can use similar steps. Since 81729 is divided by Y, we can use the inverse operation of multiplication by Y to isolate Y. So, multiplying both sides of the equation by Y gives:

[tex]81729 = 898.12 \times Y[/tex]

Simplifying the right side of the equation gives:

[tex]Y = 81729 / 898.12[/tex]

[tex]Y = 90.98[/tex]

Therefore, Y is 90.98.

To know more about equations visit :

brainly.com/question/29657988

#SPJ1

Please help!
The object above is symmetrical through Z. If Y = 11 inches, Z = 13 inches, and H = 6 inches, what is the area of the object?
A. 6.5 square inches
B. 78 square inches
C. 31 square inches
D. 156 square inches

Answers

the  correct area of the symmetrical object is option (B). 78 square inches.

Definition of Symmetry

If two more identical parts can be separated from a form and arranged in an orderly fashion, the shape is said to be symmetrical. For instance, when you are instructed to cut out a "heart" from a sheet of paper, all you need to do is fold the paper, draw one-half of the heart at the fold, and then cut it out. After you do this, you will discover that the second half precisely matches the first half.

In the first part of the object

Area of the object=½×H×Z

=½×13×6

=39 square inches

Given that object above is symmetrical through Z.

So, the area of 2nd part of object will also be 39 square inches

Hence total area is 78 square inches.

to know more about area, visit:

https://brainly.com/question

#SPJ1

round 0.956 to one decimal place

Answers

Answer: 0.96

Step-by-step explanation:

To round 0.956 to one decimal place, we need to look at the second decimal place (the hundredths place), which is 5. Since 5 is greater than or equal to 5, we need to round up the first decimal place (the tenths place), which is 9. Therefore, the rounded number to one decimal place is:

0.956 rounded to one decimal place = 0.96

When we round a number, if the digit immediately to the right of the one we want to keep is 5 or greater, we round up the digit we want to keep. In this case, the digit to the right of 5 is 6, which is greater than 5. Therefore, we round up the 5 to 6, giving us 0.96.

However, if we were instead asked to round to the nearest whole number, then we would round 0.956 up to 1.0. This is because we would look at the digit immediately to the right of the decimal point, which is 5, and since 5 is greater than or equal to 5, we would round up to the nearest whole number, which is 1.0.

At which values in the interval [0, 2π) will the functions f (x) = 2cos2θ and g(x) = −1 − 4cos θ − 2cos2θ intersect?
a: theta equals pi over 3 comma 4 times pi over 3
b: theta equals pi over 3 comma 5 times pi over 3
c: theta equals 2 times pi over 3 comma 4 times pi over 3
d: theta equals 2 times pi over 3 comma 5 times pi over 3

Answers

The values in the interval [0, 2π) for which the two points would intersect as required is; Choice C; theta equals 2 times pi over 3 comma 4 times pi over 3.

What values of θ make the two functions intersect?

Recall from the task content; the given functions are;

f (x) = 2cos2θ and g(x) = −1 − 4cos θ − 2cos2θ

Therefore, for intersection; f (θ) and g(θ):

2 cos²θ = −1 − 4cos θ − 2cos²θ

4cos²θ + 4cosθ + 1 = 0

let cos θ = y;

4y² + 4y + 1 = 0

y = -1/2

Therefore; -1/2 = cos θ

θ = cos-¹ (-1/2)

θ = 2π/3, 4π/3.

Ultimately, the correct answer choice is; Choice C; theta equals 2 times pi over 3 comma 4 times pi over 3.

Read more on trigonometry;

https://brainly.com/question/24334139

#SPJ1

1. If f = {(0,2), (-3,2), (2,5)} and g = {(3,4), (1,5), (-1,2)}, Find: f+g

Answers

Answer:

Step-by-step explanation:

F = (-1,9)

G = (-3,11)

What is 0.83333333333 as a fraction?

Answers

Answer: 41666666669 / 50000000003

Step-by-step explanation:

Calvin ordered scissors. Each pair of scissors weighs 0. 34 kg. A box of scissors weighs
64. 6 kg. How many pairs of scissors come n each box?

Answers

190 pairs of scissors came in each box.

This is an example of a word problem. As a model, a word problem depicts circumstances from the actual world that may be converted into phrases.

Weight of each pair of scissors = 0.34 kg

Weight of a box of scissors = 64.6 kg

Therefore, pairs of scissors in each box = Weight of a box of scissors ÷

                                                                      Weight of each pair

                                                                   = 64.6 ÷ 0.34

                                                                   = 190 pairs.

Hence, 190 pairs of scissors came in each box.

To learn more about word problems, refer to:

https://brainly.com/question/25693822

#SPJ4

Write the equation of a line that is perpendicular to y=½x - 9 and passes through the point (3, -2).

Answers

Answer:

y = - 2x + 4

Step-by-step explanation:

the equation of a line in slope- intercept form is

y = mx + c ( m is the slope and c the y- intercept )

y = [tex]\frac{1}{2}[/tex] x - 9 ← is in slope- intercept form

with slope m = [tex]\frac{1}{2}[/tex]

given a line with slope m then the slope of a line perpendicular to it is

[tex]m_{perpendicular}[/tex] = - [tex]\frac{1}{m}[/tex] = - [tex]\frac{1}{\frac{1}{2} }[/tex] = - 2 , then

y = - 2x + c ← is the partial equation

to find c substitute (3, - 2 ) into the partial equation

- 2 = - 2(3) + c = - 6 + c ( add 6 to both sides )

4 = c

y = - 2x + 4 ← equation of perpendicular line

Samuel bought four adult tickets to a movie for $48. Erica bought 3 adult tickets to a movie at a different theater. Erica paid $2.50 more than Samuel for each movie ticket she bought. How much did Erica spend on her movie ticket purchase?​

Answers

Answer: £43.50

Step-by-step explanation:

each ticket from samuel is £12 if erica is spending £2.50 more per ticket that is £14.50 per ticket. £14.50 x 3 = £43.50

These tables represent an exponential function. Find the average rate of
change for the interval from x = 8 to x = 9.
OA. 6561
B. 19,683
O C. 13,122
OD. 3
X
2456710
3
y
1
3
9
27
81
243
729
Interval
0 to 1
1 to 2
2 to 3
3 to 4
4 to 5
5 to 6
Average rate
of change
2
6
18
54
162
486
Jx3
Jx3
Jx3
Jx3
Jx3

Answers

The average rate of change for the interval from x = 8 to x = 9 is 13,122 (3⁸to 3⁹).

What is function?

Function is a self-contained block of code that performs a specific task. It is used to separate logical code into separate parts and make the code more manageable. A function can accept inputs, known as parameters, and return an output, known as a return value. Functions can be reused throughout a program, making the code more efficient and easier to debug.

This can be calculated by subtracting the starting value (3⁸ = 6561) from the ending value (3⁹ = 19683) and then dividing the result by the interval (9 - 8 = 1). Mathematically, this is expressed as (19683 - 6561) / (9 - 8) = 13,122. This result shows that the rate of change for the exponential function between x = 8 and x = 9 is 13,122.

To know more about function click-
https://brainly.com/question/25841119
#SPJ1

Suppose the number of water people drink in a week is normally distributed with a mean of 50 and a standard deviation of 5 glasses of water. Find the value 1 standard deviation below the mean

Answers

Answer:

Step-by-step explanation:

.

suppose that each day the price of a stock moves up 1/8th of a point with probability 1/3 and moves down 1/8th of point with probability 2/3. if the price fluctuations from one day to another are independent, what is the probability that after 6 days the stock has its original price?

Answers

After 6 days, the probability that the stock has its original price is 5/16.

There are two possible scenarios that can take place when the stock price fluctuates from one day to the next. Either the price goes up by 1/8th of a point with probability 1/3 or it goes down by 1/8th of a point with probability 2/3.

The price of the stock after six days can be denoted as S6. The price of the stock after the first day can be represented as S0.

Since the price fluctuates either up or down by 1/8th of a point on each day, the price after six days can be represented as follows:S6 = S0 + (up, up, up, up, up, up), (up, up, up, up, up, down), (up, up, up, up, down, up), ... , (down, down, down, down, down, down)

In order to return to the original price, the stock must go up and down by the same amount. As a result, there must be an equal number of ups and downs in the six-day period.

As a result, we must calculate the probability of obtaining an equal number of ups and downs over six days.

Let's represent an increase in the stock price as 'U' and a decrease as 'D.'

The total number of ways in which the stock can go up and down over six days is [tex]2^6 = 64[/tex].

The total number of ways in which the stock can return to its original price can be calculated as follows: [tex]N(UUUDDD) = 6! / (3! * 3!) = 20[/tex]

The probability of the stock returning to its original price after six days can be calculated as:

[tex]P = N(UUUDDD) / 64 = 20 / 64 = 5 / 16[/tex]

Therefore, the probability that after 6 days the stock has its original price is 5/16.

Learn more about probability here:

https://brainly.com/question/24756209

#SPJ11

5.6 Enrichment and Extension
Please answer A, B, D, F, H, l, N, and P

Answers

Step-by-step explanation:

a. f(x) = s(q(x))

b. f(x) = q(s(q(x)))

d. f(x) = g(p(h(x)))

f. f(x) = q(s(x))

i. f(x) = h(r(x))

n. f(x) = h(g(s(x)))

not sure about h and p tho

hope this helps

Match each integer with one that divides it.
18
matches
Choice
9
-14
matches
Choice
7
11
matches
Choice
11
65
matches
Choice
13

Answers

Integers are a type of number used in mathematics that represent whole numbers. They are typically denoted by the symbol Z, and can be positive, negative, or zero.

Integers do not include fractions or decimal points, so they are distinct from real numbers.

In Mathematics, integers are the collection of whole numbers and negative numbers.

18 matches 9

-14 matches 7

11 matches - 65

13 matches 65

To learn more about  "integers” refer to the : https://brainly.com/question/929808

#SPJ11

find the derivative of y equals 5 x squared sec to the power of short dash 1 end exponent (2 x minus 3 )

Answers

The derivative of the given function [tex]y = 5x^2 sec^{(-1)(2x-3)^2}[/tex] is [tex]dy/dx=-20x\sqrt{((2x-3)^2-1)}[/tex]

It can be derived as:

We can use the chain rule and the derivative of [tex]sec^{(-1)x}[/tex] which is [tex]-1/(x*\sqrt{(x^2-1)})[/tex]

First, we apply the chain rule to the function.

Let  [tex]u = (2x-3)^2[/tex], then:

[tex]y = 5x^2 sec^{(-1)u}[/tex]

[tex]dy/dx = d/dx [5x^2 sec^{(-1)u}][/tex]

[tex]dy/dx = d/dx [5x^2 sec^{(-1)[(2x-3)^2]}][/tex]

[tex]dy/dx= 5x^2 d/dx[sec^{(-1)u}][/tex]     (Using the chain rule)

Now, let [tex]v = u^{(1/2)} = (2x-3)[/tex].

Then:

[tex]dy/dx = 5x^2 d/dv [sec^{(-1)v}] dv/dx[/tex]          (Using the chain rule again)

We have:

[tex]d/dv [sec^{(-1)v}] = -1/(v*\sqrt{(v^2-1)}) = -1/[(2x-3)*\sqrt{((2x-3)^2-1)}][/tex]

Also, [tex]dv/dx = 2[/tex]

Substituting these back into the equation:

[tex]dy/dx = 5x^2 d/dv [sec^{(-1)v}] dv/dx[/tex]

[tex]dy/dx= 5x^2 (-1/[(2x-3)*\sqrt{((2x-3)^2-1)}] (2)[/tex]

Simplifying this expression gives:

[tex]dy/dx = -20x (2x-3)/[(2x-3)*\sqrt{((2x-3)^2-1)}][/tex]

[tex]dy/dx = -20x\sqrt{((2x-3)^2-1)}[/tex]

Therefore, the derivative of y with respect to x is:

[tex]dy/dx = -20x\sqrt{((2x-3)^2-1)}[/tex]

To practice more questions about derivative:

https://brainly.com/question/12047216

#SPJ11

A package is delivered 3 hours 25 minutes after it is collected, it is collected at 15:39
at what time is the package delivered

Answers

Given the data in the question we calculate that the package is delivered at 18:44.

If the package is collected at 15:39 and delivered 3 hours and 25 minutes later, we can add that amount of time to the collection time to find the delivery time.

First, we need to convert 3 hours and 25 minutes to just minutes. To do this, we multiply 3 by 60 (to convert hours to minutes) and then add 25:

3 hours and 25 minutes = (3 × 60) + 25 = 185 minutes

Now we can add 185 minutes to the collection time of 15:39:

15:39 + 185 minutes = 18:44

Therefore, the package is delivered at 18:44. The delivery time of a package is the time it takes for the package to be transported from the sender to the receiver. In this case, the package was collected at 15:39 and delivered 3 hours and 25 minutes later. To find the delivery time, we added the duration of 3 hours and 25 minutes to the collection time. It is important to keep track of delivery times to ensure timely and efficient shipping, especially for time-sensitive or perishable items. Timely delivery is crucial for businesses that rely on shipping to meet customer expectations and maintain customer satisfaction.

To learn more about amount of time click here

brainly.com/question/29823316

#SPJ4

A barista mixes 12lb of his secret-formula coffee beans with 15lb of another bean that sells for $18 per lb. The resulting mix costs $20 per lb. How much does the barista's secret-forumula means cost per pound?

Answers

Answer:

The baristas secret formula beans cost per pound is $19.2.

Step-by-step explanation:

Given is that a barista mixes 12 lb of his secret-formula coffee beans with 15 lb of another bean that sells for $18 per lb.The cost of 1 pound of another bean as -$(15/18) = $(5/6)Assume that 1 pound of secret coffee costs ${x}. So, we can write -x + 5/6 = 20x = 20 - 5/6x = 19.2Therefore, the baristas secret formula beans cost per pound is $19.2.

Answer:

Find 5 rational numbers between 4/5 - and * 3/7

The probability distribution of the amount of memory X (GB) in a purchased flash drive is given below. x 1 2 4 8 16 p(x) .05 .10 .35 .40.10 Compute the following: E(X), E(X2), V(X), E(3x + 2), E (3X² + 2), V (3x + 2), E(X +1), V(X + 1).

Answers

To solve the question asked, you can say:  Therefore, the final answers expressions are: E(X) = 5.8; E(X²) = 59.8; V(X) = 21.16 and E(3X + 2) = 20.4

what is expression ?

In mathematics, an expression is a set of numbers, variables, and mathematical operations such as addition, subtraction, multiplication, division, and exponentiation that represent quantities or values. Expressions can be as simple as "3 + 4" or as complex as they can contain functions like "sin(x)" or "log(y)" . Expressions can be evaluated by substituting values ​​for variables and performing mathematical operations in the order specified. For example, if x = 2, the expression "3x + 5" is 3(2) + 5 = 11. In mathematics, formulas are often used to describe real-world situations, create equations, and simplify complex math problems. 

To calculate these values, we first need to compute the mean (expected value) and variance of X, which are given by:

E(X) = ∑[x * p(x)]

= 1 * 0.05 + 2 * 0.10 + 4 * 0.35 + 8 * 0.40 + 16 * 0.10

= 5.8

E(X²) = ∑[x² * p(x)]

= 1² * 0.05 + 2² * 0.10 + 4² * 0.35 + 8² * 0.40 + 16² * 0.10

= 59.8

V(X) = E(X²) - [E(X)]²

= 59.8 - 5.8²

= 21.16

E(3X + 2) = 3E(X) + 2

= 3(5.8) + 2

= 20.4

E(3X² + 2) = 3E(X²) + 2

= 3(59.8) + 2

= 179.4

V(3X + 2) = V(3X)

= 9V(X)

= 9(21.16)

= 190.44

E(X + 1) = E(X) + 1

= 5.8 + 1

= 6.8

V(X + 1) = V(X)

= 21.16

Therefore, the final answers are:

E(X) = 5.8

E(X²) = 59.8

V(X) = 21.16

E(3X + 2) = 20.4

E(3X² + 2) = 179.4

V(3X + 2) = 190.44

E(X + 1) = 6.8

V(X + 1) = 21.16

To know more about expressions visit :-

https://brainly.com/question/14083225

#SPJ1

Quadrilateral KLMN has vertices at K(6, 0), L(10, -5), M(5, -9), and N(1, -4).
Is KLMN a square? Justify your answer.
Yes, all sides are congruent, and KL and LM are perpendicular.
Yes, all sides are congruent, and KL and MN are parallel.
No, KL and LM are not perpendicular.
No, KL and MN are not parallel.

Answers

We have shown that KLMN has four congruent sides and opposite sides are parallel and perpendicular, which makes KLMN a square.

What is a graph?

A diagram (such as a series of one or more points, lines, line segments, curves, or areas) that represents the variation of a variable in comparison with that of one or more other variables.

To determine if KLMN is a square, we need to check if all four sides are congruent and if opposite angles are congruent.

We can find the length of each side using the distance formula:

KL = sqrt((10 - 6)^2 + (-5 - 0)^2) = sqrt(16 + 25) = sqrt(41)

LM = sqrt((5 - 10)^2 + (-9 - (-5))^2) = sqrt(25 + 16) = sqrt(41)

MN = sqrt((1 - 5)^2 + (-4 - (-9))^2) = sqrt(16 + 25) = sqrt(41)

NK = sqrt((6 - 1)^2 + (0 - (-4))^2) = sqrt(25 + 16) = sqrt(41)

So all four sides have the same length of sqrt(41), which is a necessary condition for a square.

To check if opposite angles are congruent, we can find the slope of each pair of opposite sides:

slope KL = (-5 - 0)/(10 - 6) = -5/4

slope MN = (-4 - (-9))/(1 - 5) = 5/4

slope LM = (-9 - (-5))/(5 - 10) = -4/5

slope NK = (0 - (-4))/(6 - 1) = 4/5

We can see that the slopes of KL and MN are negative reciprocals, as are the slopes of LM and NK. This means that opposite sides are parallel and perpendicular, respectively, which is another necessary condition for a square.

Learn more about graphs on:

https://brainly.com/question/19040584

#SPJ1

We have demonstrated that KLMN is a square since it has four congruent sides and opposite sides that are parallel and perpendicular.

What is graph?

A visual representation of the variation of one variable with relation to one or more other variables, such as a collection of dots, lines, line segments, curves, or regions.

To determine if KLMN is a square, we need to check if all four sides are congruent and if opposite angles are congruent.

We can find the length of each side using the distance formula:

KL = sqrt((10 - 6)² + (-5 - 0)²) = sqrt(16 + 25) = sqrt(41)

LM = sqrt((5 - 10)² + (-9 - (-5))²) = sqrt(25 + 16) = sqrt(41)

MN = sqrt((1 - 5)² + (-4 - (-9))²) = sqrt(16 + 25) = sqrt(41)

NK = sqrt((6 - 1)² + (0 - (-4))²) = sqrt(25 + 16) = sqrt(41)

So all four sides have the same length of sqrt(41), which is a necessary condition for a square.

To check if opposite angles are congruent, we can find the slope of each pair of opposite sides:

slope KL = (-5 - 0)/(10 - 6) = -5/4

slope MN = (-4 - (-9))/(1 - 5) = 5/4

slope LM = (-9 - (-5))/(5 - 10) = -4/5

slope NK = (0 - (-4))/(6 - 1) = 4/5

We can observe that the slopes of KL, MN, LM, and NK are all negative reciprocals. Another prerequisite for a square is that the opposing sides must be parallel and perpendicular, respectively.

Learn more about graphs on:

brainly.com/question/19040584

#SPJ1

Reduce each expression to a polynomial

((y-b)^(2))/(y-b+1)+(y-b)/(y-b+1)

Answers

The given expression ((y-b)²/(y-b+1)+(y-b)/(y-b+1) after being reduced to a polynomial, can be represented as y-b.

In order to reduce the given equation to a polynomial, we are required to simplify and combine like terms. First, we can simplify the expression in the numerator by expanding the square:

((y-b)²/(y-b+1) = (y-b)(y-b)/(y-b+1) = (y-b)²/(y-b+1)

Now, we can combine the two terms in the equation by finding a common denominator:

(y-b)²/(y-b+1) + (y-b)/(y-b+1) = [(y-b)² + (y-b)]/(y-b+1)

Next, we can combine the terms in the numerator by factoring out (y-b):

[(y-b)² + (y-b)]/(y-b+1) = (y-b)(y-b+1)/(y-b+1)

Finally, we can cancel out the common factor of (y-b+1) in the numerator and denominator to get the polynomial:

(y-b)(y-b+1)/(y-b+1) = y-b

Therefore, the equation ((y-b)²)/(y-b+1)+(y-b)/(y-b+1)  after being simplified, is equivalent to the polynomial y-b.

Learn more about polynomials :

https://brainly.com/question/30127172

#SPJ4

Other Questions
TRUE/FALSE. Emotional appeals tend to lose their effectiveness when they take into account the values, motives, and goals of group members. How can a change in a DNA nucleotide lead to a change in the protein? choose all the shapes with at least one pair of perpendicular sides Which of these could be possible if a single nucleotide in the regulatory sequence of DNA controlling transcription was changed? On June 1, Sawyer Co. borrowed $5,000 cash from Crystal Bank by signing a 45-day, 12% interest-bearing note. On July 16, Sawyer pays the amount due in full. Sawyer would record this payment with a ______ to interest Expense in the amount of __________. Simplify (cos^2a - cot^2a)/(sin^2a - tan^2a) FILL IN THE BLANK. _______ is NOT given by Rae as a reason the Christian worldview clarifies a virtuous life the demand factor involves households, businesses, and government purchasing the economy's expanding output of goods and services so that there will be . multiple choice question. A. no unplanned increase in inventories and resources will remain fully employed zero B. increase in inventories and resources will remain constant C. unplanned decreases in inventories and resources will be overemployed unplanned D. increases in inventories and resources will be unemployed Question 14 Previous Next> The industry-low, industry-average, and industry-high cost benchmarks that appear on p. 6 and p. 7 of each issue of the Footwear Industry Report o are important enough to always merit attention by your company's managers, this is because when the benchmarks for one or more measures reveal that your company's outcome(s) were too far out-of-line and almost certainly impaired/weakened your company's overall performance, then your management team is well-advised to consider taking corrective action in the next decision round are of considerable value to the managers of companies pursuing a low-cost strategy but are of very limited value to company managers employing other types of strategies. are most valuable to the managers of companies whose cost benchmarks are above the industry-average benchmarks and/or who are looking for evidence to confirm a suspected need to secure more celebrity endorsements in the upcoming decision round and steal branded market share away from rivals in one or more regions are of little value because the benchmarking data do not identify which companies have the lowest/highest costs for any of the reported cost benchmarks. are of considerable value to the managers of companies whose prior-year EPS was below the industry-average benchmark in one of more geographic regions. FILL IN THE BLANK. Intimacy is the desire for _____.A. social connectionB. generativityC. sexual intercourseD. self-esteem the beneficial effects of belonging to a nation in nineteenth-century europe included access to which of the following?a. productive landbetter schoolsb. efficient bureaucraciesc. commercial markets WILL GIVE BRAINLIEST ANSWER AND 5 STAR RATING HELP ASAP if a pipet bulb contains 5 ml of hydrogen gas, how many mL of oxygen gas would be needed to make the optimum mixture? if the area of the quadrilateral ABCS is 924cm^2 and the length of the diagonal AC is 33cm,find the sum of lengths of the perpendicular from points B and D to AC.please answer with full steps asap Question 25 (2 points)Suppose the Math Department has 17 full-time faculty members. If 3 are selected toattend a conference in Las Vegas, in how many different ways can you selected the 3individuals?3176804080 A good strategy for weight gain in underweight individuals is to ____. a. eat energy-dense foods b. select foods with high levels of trans fats Sammy, a 9-year-old girl, is overweight. She is constantly teased and tormented by other children in school. Her parents try to cheer her up by telling her that they were also overweight when they were young and even now they are overweight but they don't worry about what others say about them. Based on the given information, which of the following is the primary cause for Sammy being overweight? The residual interest in a corporation belongs toA. the common stockholders.B. the preferred stockholders.C. the Board of Directors.D. Management. what information is needed to balance a chemical formula equation example periodic table or list of chemicals A firm will expand the amount of output it produces as long as itsA. average total revenue exceeds its average total cost.B. average total revenue exceeds its average variable cost.C. marginal cost exceeds its marginal revenue.D. marginal revenue exceeds its marginal cost. Justin has a personal library with over 5,000 novels, most of them bought from Barnes and Noble. He is an example of a Barnes and Noble____